0 Daumen
491 Aufrufe

Guten Tag,

Hier noch eine weiter Aufgabe, welche eine weiter Teilaufgabe der Aufgabe und meiner vorherig gestellten Frage ist.

A728173B-0603-414B-8C46-FA200F42C244.jpeg

Text erkannt:

(b) Zeigen Sie, dass
\( \frac{1}{\sqrt{n}}\|\underline{x}\| \leq\|\underline{x}\|_{\infty} \leq\|\underline{x}\| \)
für alle \( \underline{x} \in \mathbb{R}^{n} \) gilt, wobei \( \|\cdot\| \) die euklidische Norm auf dem \( \mathbb{R}^{n} \) bezeichne.

Wie beweise ich diese Ungleichung? Benutzt man für den Beweis die vollständige Induktion oder muss man sich eines anderen Werkzeuges bemächtigen?

Avatar von

Wie sind die Normen definiert? $$ \| x \| = \sqrt{ \sum_{k=1}^n x_k^2  } $$ und $$ \| x \|_\infty = \max_{k=1..n} x_k $$

Ja genau so sind die Normen definiert.

1 Antwort

0 Daumen

Aloha :)

Es ist \(\vec x=(x_1,x_2,\ldots,x_n)\in\mathbb R^n\) gegeben.

Wir wählen aus den Komponenten eine mit maximalem Betrag und nennen sie \(x_m\):$$|x_k|\le|x_m|\quad\text{für alle }k=1,2,\ldots,n\quad;\quad m\in\{1,2,3,\ldots,n\}$$Damit können wir folgende Ungleichungskette aufstellen:$$\frac{1}{\sqrt n}\left\|\vec x\right\|=\frac{1}{\sqrt n}\underbrace{\sqrt{x_1^2+x_2^2+\cdots+x_n^2}}_{\text{n Summanden}}\le\frac{1}{\sqrt n}\sqrt{x_m^2+x_m^2+\cdots+x_m^2}$$$$\phantom{\frac{1}{\sqrt n}\left|\vec x\right\|}=\frac{1}{\sqrt n}\sqrt{n\cdot x_m^2}=\sqrt{x_m^2}=\left|x_m\right|=\left\|\vec x\right\|_\infty$$$$\left\|\vec x\right\|_\infty=\left|x_m\right|=\sqrt{x_m^2}\le\underbrace{\sqrt{x_1^2+x_2^2+\cdots+x_1^2+\cdots+x_n^2}}_{\text{einer der Indizes ist gleich m}}=\left\|\vec x\right\|$$

Avatar von 148 k 🚀

Steht m in diesem Falle für den Index des Größten Betrages aus dem Vektor? Bzw. gibt m die Komponente aus dem Vektor an, bei dem der Betrag maximal ist?

Ja, genau. Es muss ja eine vom Betrag her größte Komponente unter den \(x_k\) geben. Diese habe ich \(x_m\) genannt, das "m" soll "maximal" bedeuten.$$x_m\coloneqq\max\limits_{k=1,\ldots,n}\left|x_k\right|$$

Ich würde mit den Quadraten der Normen arbeiten. Dann wird Manches

einfacher / übersichtlicher ;-)

Okay gut ,dann habe ich das schonmal verstanden. Aber eine weitere Frage habe ich noch, wie kommt man von 1/√n ·√n·xm2 auf xm2?

@ermanus Meinst du weil sich dann auch die Wurzeln alle auflösen bzw. verschwinden?

$$\frac{1}{\sqrt n}\cdot\sqrt{n\cdot x_m}=\frac{1}{\sqrt n}\cdot\sqrt{n}\cdot\sqrt{x_m}=\frac{\sqrt n}{\sqrt n}\cdot\sqrt{x_m}=\sqrt{x_m}$$

Achsooo, okay. Wirklich vielen vielen Dank für die Hilfe:)

"Meinst du weil sich dann auch die Wurzeln alle auflösen bzw. verschwinden?"

Ja. Dann ist alles "angenehm wurzelfrei". Das ändert aber prinzipiell

nichts an der schönen Lösung von Tschakabumba.

Ein anderes Problem?

Stell deine Frage

Willkommen bei der Mathelounge! Stell deine Frage einfach und kostenlos

x
Made by a lovely community